Answer:
27 children
Step-by-step explanation:
So, if you add 9 +4 = 13. 13 x 3 = 39.
3 times 9 = 27
Hope this helped! Pls mark me brainliest!
Answer:
27 children
Step-by-step explanation:
9+4=13
39/13=3
9x3=27
Find each product or quotient. √x³ / √5 x²
The quotient of √x ³ / √5 x ² is √ ( x / 5 )
To find : the product of √x ³ / √5 x ²
By checking the power of roots for each term
The terms which are given are √x ³ and √5 x ²
Since both the numbers have the same root power they can be combined under the same square root for further operation
Dividing the terms
=> √ ( x ³ / 5 x ² )
=> √ ( x / 5 )
Therefore, we get the quotient of √x ³ / √5 x ² as √ ( x / 5 ).
Learn more about Roots here:
https://brainly.com/question/16932620?referrer=searchResults
#SPJ9
We are given that Ray E B bisects ∠AEC. From the diagram, ∠CED is a right angle, which measures
The solutions to the given gaps in the angle proofs are;
∠CED is a right angle, and measures 90°
The measure of angle AEC must also be 90° by the Angle addition postulate
How to complete angle proof?We are told that the Ray EB bisects ∠AEC.
Now, we are told that angle ∠CED is a right angle, and as such by definition of right angles it measures 90°
Now, Since the measure of a straight angle is 180°, the measure of angle AEC must also be 90° by the Angle addition postulate. This is because angle AEC and angle CED sum up to 180°.
Thus, we conclude that the solutions to the given gaps are;
∠CED is a right angle, and measures 90°
The measure of angle AEC must also be 90° by the Angle addition postulate
Read more about Angle Proofs at; https://brainly.com/question/11891208
#SPJ1
Complete question is;
Complete the paragraph proof. We are given that Ray E B bisects ∠AEC. From the diagram, ∠CED is a right angle, which measures degrees. Since the measure of a straight angle is 180°, the measure of angle must also be 90° by the . A bisector cuts the angle measure in half. m∠AEB is 45°.
Aaron is designing a party game in which he needs exactly 24 possible outcomes. which sets of actions can he use to have a statistically fair game?
If Aaron is designing a party game in which he needs exactly 24 possible outcomes, then he can use the following sets of actions to have a statistically fair game from the Statistics point of view,
Toss a coin twice, and then then roll a 6-sided number cube.
How can he play a statistically fair game to get exactly 24 outcomes?
The game has 24 alternative outcomes, as far as we know. The next step is to identify a series of activities that produces 24 distinct outcomes with equal probabilities.
The first of the alternatives is the only one with a sample space of 24 elements.
Flip two coins, then roll a D6.
The results of each coin toss are 2: (tails and heads).
There are 6 outcomes for the D6: (1, 2, 3, 4, 5, 6)
The combined outcomes of the two throws and rolling the number are then given by the product between the numbers of outcomes for each individual part, as solved below:
C = 2 × 2 × 6
C = 24
Thus, by tossing a coin twice, and then then rolling a 6-sided number cube, Aron can play a statistically fair game if h needs exactly 24 outcomes.
Learn more about outcomes here:
https://brainly.com/question/9222927
#SPJ1
A rectangular lawn is 100m long and 45m wide.
There are 3 circular ponds, with diameters of 20m, 10m and 5m respectively.
Mrs Jones wants to cover the lawn with grass seed.
Each packet of grass seed covers 50m² and costs £1.49
How much will it cost Mrs Jones to cover the lawn with grass seed?
Answer:
The area of the rectangular lawn is 100m long and 45m wide, so it has an area of 100 * 45 = <<10045=4500>>4500m².
The three circular ponds have diameters of 20m, 10m, and 5m, so their areas are 314.16, 78.54, and 19.63 square meters respectively.
The total area of the ponds is 314.16 + 78.54 + 19.63 = <<314.16+78.54+19.63=412.33>>412.33 square meters.
The total area of the lawn that needs to be covered with grass seed is 4500 - 412.33 = <<4500-412.33=4087.67>>4087.67 square meters.
Each packet of grass seed covers 50 square meters, so Mrs Jones will need 4087.67 / 50 = <<4087.67/50=81.75>>81.75 packets of grass seed.
Each packet costs £1.49, so the total cost to Mrs Jones will be 81.75 * 1.49 = £<<81.751.49=121.78>>121.78. Answer: {121.78}.
Step-by-step explanation:
Please help please I need it now please
Solve for x in the following problem: 9 = 3/5x
Answer:
dont know exactly what way ur studying to solve for it rn but heres a simple answer and if theres a diff process yall r using then ermmm sorry bud
x= 1/15
Answer:
x = 15
Step-by-step explanation:
\(\sf 9 = \dfrac{3}{5}x\\\\ \text{Multiply both sides by 5},\\\\9*5 = \dfrac{3}{5}*5*x\\\\ 9 *5 = 3x\\\\\text{Divide both sides by 3}\\\\\dfrac{9*5}{3}=x\\\\3*5=x\\\\\boxed{\bf x = 15}\)
40 percent in to fraction decmal
Answer:
0.4 hope this helps
Step-by-step explanation:
Answer:
Well 40 percent is 2/5 as a fraction and 2/5 as a decimal is 0.4
Step-by-step explanation:
through: (-1,0), parallel to y = 4x
Answer:
Find the slope of the original line and use the point-slope formula y−y1=m(x−x1)to find the line parallel to y=4x.
y=4x+4
Step-by-step explanation:
Herschel uses an app on his smartphone to keep track of his daily calories from meals. One day his calories from breakfast were 115 more, than his calories from lunch, and his calories from dinner were 200 less than twice his calories from lunch. If his total caloric intake from meals was 2135, determine his calories for each meal.
Answer:
calories
Breakfast: 670
Lunch: 555
Dinner: 910
Total = 2135 calories
Step-by-step explanation:
Let B, L and D, stand for the calories from Breakfast, Lunch, and Dinner
One day his calories from breakfast were 115 more, than his calories from lunch:
B = L + 115
his calories from dinner were 200 less than twice his calories from lunch.:
D = 2L - 200
total caloric intake from meals was 2135:
B + L + D = 2135
------
We have B and D defined in terms of L from the first two equations, so use them in the third:
B + L + D = 2135
[L+115] + L + [2L-200] = 2135
4L - 85 = 2135
4L = 2220
L = 555 calories
B = L + 115
B = 555 + 115
B = 670 calories
D = 2L - 200
D = 2(555) - 200
D = 910 calories
================
CHECK
Does B + L + D = 2135 calories?
670 + 555 + 910 = 2135?
2135 = 2135? YES
(4+5)(4-51) =
please help i don’t feel like doing math rn
Answer:
423
Step-by-step explanation:
Answer:
-423
Step-by-step explanation:
Hope This Helps!
Plz Mark Brainliest!
The future of news? virtual reality de la pena has worked in multiple media in terms of journalism. which has had the most significant impact on her? why do you think so?
Virtual reality has had the most significant impact on de la Pena's journalism career due to its ability to provide a more immersive and engaging news experience and its potential to reach a wider audience.
The question is about the future of news and the impact of virtual reality on de la Pena's journalism career.
In terms of the most significant impact on de la Pena's career, it can be argued that virtual reality has had the most significant impact. This is because virtual reality has revolutionized the way news is presented and consumed.
Virtual reality allows journalists like de la Pena to immerse their audience in a story by providing a more immersive and engaging experience. By using virtual reality, de la Pena has been able to create powerful and impactful stories that allow the audience to experience events and situations firsthand. This has the potential to create a deeper understanding and empathy among viewers.
Furthermore, virtual reality has also expanded the reach of journalism by allowing audiences from different parts of the world to experience stories that they wouldn't have been able to otherwise. This has the potential to foster a more global perspective and increase awareness of important issues.
Overall, virtual reality has had the most significant impact on de la Pena's journalism career due to its ability to provide a more immersive and engaging news experience and its potential to reach a wider audience.
To know more about journalism refer here:
https://brainly.com/question/20883382
#SPJ11
need help with math thanks
Answer:
Use mathematical skills to acquire the correct answer of 7.
Step-by-step explanation:
Hee hee hee haw.
Answer:
7 is the answer
Step-by-step explanation:
3x + 8 = 29
or, 3x + 8 - 8 = 29 - 8
or, 3x = 21
or, 3x/3 = 21/3
or, x = 7
anova’s are used when the study has: three or more groups to compare one or more groups to compare four or more groups to compare five or more groups to compare
ANOVA is generally used when a study has three or more groups to compare, but it can also be applied to situations with fewer than three groups
ANOVA (Analysis of Variance) is a statistical test used to analyze the differences between means when comparing two or more groups. The specific number of groups required for using ANOVA depends on the research question and design of the study.
In general, ANOVA is commonly used when there are three or more groups to compare. It allows for the examination of whether there are statistically significant differences between the means of these groups.
This can be useful in various research scenarios where multiple groups are being compared, such as in experimental studies with different treatment conditions, or in observational studies with multiple categories or levels of a variable.
However, it is important to note that ANOVA can also be used when there are only two groups, although a t-test may be more appropriate in such cases.
On the other hand, there is no inherent restriction on the maximum number of groups for conducting an ANOVA. It can be used when comparing four, five, or even more groups, as long as the necessary assumptions of the test are met and the research question warrants the comparison.
To know more about ANOVA refer to
https://brainly.com/question/31809956
#SPJ11
The price of one pen or 2 copies is Rs. 40. What is the price of 5 pens and 20 copies ?
The price of one pen or 2 copies is Rs. 40. The price of 5 pens and 20 copies is Rs. 600.
Given, that the price of 1 pen or 2 copies is Rs.40. Therefore, we have the price of one pen to be 40 or 2 copies to be 40.
Thus, the price of one pen is Rs.40.
The price of one copy is 40/2 = Rs.20.
Therefore,
Price of 5 pens is = 40*5 = Rs.200
Price of 20 copies is = 20*20 = Rs.400
The total price of 5 pens and 20 copies is = 200+400 = Rs.600.
Therefore, the total price is Rs.600.
What are Comparing quantities?
Comparing quantities is the process of finding an unknown quantity which has a relationship with a given condition related to it.We will be given one or more quantities with their units and asked to find the value of the same quantity for different units (i.e. in higher values or in lower values).The standard method of solving is to give some numbers of quantity with its unit we find the value of one quantity of that unit and then proceed with the asked amount of quantity with the given unit.Here, quantity refers to any item, factors, functions etc...To learn more about Comparing quantities visit:
https://brainly.com/question/12472221
#SPJ9
The terminal side of ∅ in standard position contains the point (-4, -2). Find the exact value for cot∅.
A. -2
B. -1/2
C. 2
D. 1/2
====================================================
Work Shown:
cot(theta) = cos(theta)/sin(theta)
cot(theta) = x/y
cot(theta) = -4/(-2)
cot(theta) = 2
4.4.24
Question Help
Let AABC ADEF. Find mZE.
The value of mZE is 19
(Simplify your answer. Type an integer or a decimal.)
(83+y)
2x+63
С
6x +3
(49-23)
E
Help please
Answer:
The angle marked E is 120 degrees
Step-by-step explanation:
Here, we want to get the value of the angle marked E
when two triangles are similar/congruent, we have equal angle measures
Using the markings, we can deduce the following relationships;
4y -28 = 83 + y
4y-y = 83 + 28
3y = 111
y = 111/3
y = 37
So, we have;
4y-28 as
4(37) - 28
= 120
An art contest has 12 entries. Prizes are being awarded for the first and second
place winners. How many different ways can the prizes be given?
Answer:
132.
Step-by-step explanation:
There are a total of 12 entries.
A total of 12 people can be awarded first place. After first place has been awarded, there are 11 people remaining.
\(12*11 = 132.\)
Therefore, there is a total of 132 different ways the prizes can be given.
Dos personas reciben a los carros que entran a un estacionamiento. La primera persona entrega un boleto verde cada dos carros que entran. La segunda persona entrega un boleto azul cada tres carros que entran. ¿Qué número ocupará en la fila el tercer carro que recibirá boletos de ambos colores?
Por lo tanto, el tercer carro que recibirá boletos de ambos colores ocupará la posición número 18 en la fila.
Hola, entiendo que quieres saber en qué posición de la fila se encontrará el tercer carro que recibirá boletos de ambos colores (verde y azul). Para esto, vamos a analizar la situación:
- La primera persona entrega un boleto verde cada 2 carros.
- La segunda persona entrega un boleto azul cada 3 carros.
Un carro que recibe boletos de ambos colores será aquel que ocupa una posición que es múltiplo común de 2 y 3. El mínimo común múltiplo (MCM) de 2 y 3 es 6. Por lo tanto, cada 6 carros, habrá uno que reciba boletos de ambos colores.
Para encontrar el tercer carro que recibirá boletos de ambos colores, simplemente multiplicamos el MCM (6) por la cantidad de carros que buscamos (3):
6 × 3 = 18
Por lo tanto, el tercer carro que recibirá boletos ición número 18 en la fila.
To learn more about encontrar, refer below:
https://brainly.com/question/30634577
#SPJ11
find a cartesian equation for the curve and identify it. r = 2 csc(θ)
The cartesian equation of the curve \(r=2 \hspace{0.1cm} csc \hspace{0.1cm} \theta\) is \(y=2\).
A Cartesian equation is essential in mathematics. It corresponds to a mathematical formula that expresses the connection between elements as a function of their positions on a plane known as Cartesian.
A two-dimensional coordinate scheme called the Cartesian plane employs a horizontal x-axis and an upward y-axis to identify locations in space.
Given that, \(r=2 \hspace{0.1cm} csc \hspace{0.1cm} \theta\).
So, \(csc\hspace{0.1cm}\theta=cosec \hspace{0.1cm}\theta\).
The equation becomes as follows:
\(r=2cosec\hspace{0.1cm} \theta\)
By using the trigonometric equation \(cosec\hspace{0.1cm} \theta=\frac{1}{sin\hspace{0.1cm}\theta}\), we get
\(r= \frac{2}{sin \hspace{0.1cm} \theta}\)
Multiplying both sides by \(sin\hspace{0.1cm}\theta\), we get
\(r \hspace{0.1cm}sin\hspace{0.1cm}\theta =2\hspace{0.1cm}\frac{sin\hspace{0.1cm}\theta}{sin\hspace{0.1cm}\theta}\)
\(rsin\hspace{0.1cm}\theta=2\)
By the parametric equations \(x=rcos\hspace{0.1cm}\theta\) and \(y=rsin\hspace{0.1cm}\theta\), we get
\(y=2\)
It is a horizantal line.
Hence, the cartesian equation of the curve \(r=2 \hspace{0.1cm} csc \hspace{0.1cm} \theta\) is \(y=2\).
Read more about cartesian equation:
https://brainly.com/question/30857232
The cartesian equation for the curve is: y = 2 This is a horizontal line passing through the point (0,2).
To find a Cartesian equation for the curve given by the polar equation r = 2 csc(θ), we will convert the polar coordinates (r, θ) into Cartesian coordinates (x, y) using the following relationships:
x = r * cos(θ)
y = r * sin(θ)
Step 1: Express r in terms of θ
r = 2 csc(θ)
Step 2: Since csc(θ) = 1 / sin(θ), rewrite the equation as
r = 2 / sin(θ)
Step 3: Express x and y in terms of r and θ
x = r * cos(θ)
y = r * sin(θ)
Step 4: Substitute r from Step 2 into the y equation
y = (2 / sin(θ)) * sin(θ)
Step 5: Simplify the equation
y = 2
The Cartesian equation for the given polar equation is y = 2, which represents a horizontal line passing through the point (0, 2).
Learn more about cartesian equation:
brainly.com/question/11676110
#SPJ11
0
Solve 3 (2x-- 1) + 5 (x+2) = 7(x-3)
Answer:
10x-15+7x-21
Step-by-step explanation:
Take parenthesis out
5x-5+5x+10=7x-21
10x-15+7x-21
regular pentagon and rectangle have the same perimeter. The length of the rectangle and the side of the pentagon are equal. The width is 10 more than the length. Write an equation that describes the scenario.
pentagon’s perimeter:
rectangle’s perimeter:
scenario’s equation:
Answer: pentagons perimeter: 5l rectangles perimeter: 2l+ 2(l+10) scenarios equation; 5l=2l+2(1+10)
Step-by-step explanation:
Answer:
pentagon: second choice
rectangle: fourth choice
scenario: fourth choice
Step-by-step explanation:
Need help pls I’m begging u
Answer:
D (last one), \(x^2+y^2=45\)
Step-by-step explanation:
We need to find the distance between (0,0) and (-3,6). Using the distance formula we get:
\(d=\sqrt{\left(x_2-x_1\right)^2+\left(y_2-y_1\right)^2}\)
\(d=\sqrt{\left(-3-0\right)^2+\left(6-0\right)^2}\)
\(d=3\sqrt{5}\)
Since equation of circle is: \(x^2+y^2=r^2\) we have to square the distance we just found, which gives us 45.
Answer:
x^2 + y^2 = 45
Step-by-step explanation:
(x − a ) ^2 + ( y − b ) ^2 =r^2
= (-3-0)^(2)+(6-0)^(2)
=45
does anyone know to solve this
Answer:
slope =x2-x1/y2-y1
= 0-6/4-0
=-6/4
What is the value of m that makes the equation true?
5^m x (5^-7) =5^12
Answer:
the answer is m=19
Step-by-step explanation:
5^(m-7)=5^12
m-7=12(5 is cancelled because it is common in both sides)
m=19
If B is the midpoint of AC, AB = 8x + 2 and BC = 5x + 14, then what is the length of AB?
Answer:
The length of AB: 34
Step-by-step explanation:
What are we given?
B = Midpoint of AC
AB = 8x + 2
BC = 5x + 14
|--------------|--------------|
A B C
(This is what your line would look like)
Our equation would be:
AB + BC = AC
(8x + 2) = (5x + 14)
(AB) = (BC)
(Assuming that our line and points are evenly spread out, we can say that they can equal each other)
8x + 2 = 5x +14
- 5x- 2 = -5x -2
---------------------
3x = 12
\(\frac{3x}{3}=\frac{12}{3}\)
---------------------
x = 4
1) Substitute
(8x + 2) + (5x + 14) = AC
(AB) + ( BC) = AC
AB = 8(4) + 2
32 + 2
= 34
BC = 5(4) + 14 =
20 + 14
= 34
(We may be able to assume that since the length of the line is possible the same for AB and BC this is considered correct)
The length of the line segment AB is 34 units.
Given that, B is the midpoint of AC, AB = 8x + 2 and BC = 5x + 14.
What is the midpoint of line segment?A midpoint of a line segment is the point on a segment that bisects the segment into two congruent segments. The midpoint of a line segment is the point on a segment that is at the same distance or halfway between the two ending points.
Since, B is the midpoint of AC
AB=BC
8x + 2=5x+14
Subtract 5x on both sides of the equation, that is
8x+2-5x=5x+14-5x
⇒ 3x+2 = 14
Transpose 2 to other side of the equation, we get
3x = 14 - 2
⇒ 3x=12
Transpose 3 to other side of the equation, we get
x=4
So, AB=8x+2
=8×4+2
=32+2=34 units
Therefore, the length of the line segment AB is 34 units.
To learn more about the line segment visit:
https://brainly.com/question/25727583.
#SPJ5
Fifty two is the sum of six and the product of a number and two. What is the number?
Help me pls, I’m so dumb .
Answer:
y=2/3x
Step-by-step explanation:
Answer:
1.5
Step-by-step explanation:
c=Y/X
6/4=1.5
12/8=1.5
15/10=1.5
im totally NOT giving away points and this totally IS a real question so pls help (jkjk)
Answer:
Super Hyper Ultra Ultimate Deluxe Perfect Amazing Shining God 東方不敗 Master Ginga Victory Strong Cute Beautiful Galaxy Baby 無限 無敵 無双 senchou here, thank you for the points :D
Step-by-step explanation:
a ladder 30m long rest on a vertical wall. if the angle make an angle of 63°with the wall find the distance between the foot of the ladder and the wall
If the coefficient of determination is .90, the percentage of variation in the dependent variable explained by the variation in the independent variable is a..81%. b..90%. c.81%. d.90%.
The variation in the independent variable is 0.81%. Thus option C is correct option.
According to the statement
We have given that the coefficient of determination is .90, and we ahve to find the variation in the independent variable.
So, For this purpose, we know that the
An independent variable is the variable you manipulate, control, or vary in an experimental study to explore its effects.
And
We know that the
The independent variable is the cause. Its value is independent of other variables in your study. The dependent variable is the effect. Its value depends on changes in the independent variable.
Due to this reason the variation in the independent variable is 0.81%.
So, The variation in the independent variable is 0.81%. Thus option C is correct option.
Learn more about independent variable here
https://brainly.com/question/25223322
#SPJ4